抱歉,您的浏览器无法访问本站
本页面需要浏览器支持(启用)JavaScript
了解详情 >

一方の笔记本

The only source of knowledge is experience.

这篇博客用于记录一些比较有趣的积分问题。

\(\displaystyle I = \int \sin(101x)(\sin x)^{99} dx\)

\[\displaystyle I = \int \sin(101x)(\sin x)^{99} dx = \int (\sin(100x)\cos x + \sin x \cos (100x) )(\sin x)^{99}dx\]

\[=\frac{1}{100}\int \sin(100x) d(\sin x)^{100} + \int (\sin x)^{100} \cos(100x) dx = \sin(100x) (\sin x)^{100} + C \]

\(\displaystyle I = \int \frac{1+\sin x}{1 + \cos x}e^x dx\)

\[\displaystyle I = \int \frac{e^x}{1+ \cos x} dx + \int \frac{\sin x (1 - \cos x)}{(1+\cos x)(1- \cos x)} e^x dx = \int \frac{e^x}{1+ \cos x} dx + \int \frac{1 - \cos x}{\sin x} e^x dx\]

又有:

\[\int \frac{1 - \cos x}{\sin x} e^x dx = \int \frac{2 \sin^2 \frac{x}{2}}{2 \sin\ \frac{x}{2} \cos \frac{x}{2}} e^x dx = \int e^x \tan\frac{x}{2} dx = e^x \tan \frac{x}{2} - \int \frac{e^x}{1+ \cos x} dx\]

因此\(I \displaystyle = e^x\tan\frac{x}{2} + C\)

\(\displaystyle I = \int_{0}^{+\infty} \frac{\sin^3 x}{x^3} dx\)

\[\displaystyle \sin 3x = 3\sin x - 4\sin^3 x \Rightarrow \sin^3 x = \frac{3\sin x - \sin 3x}{4}\]

\[I = \frac{\sin^3 x}{x^3}dx = \frac{1}{4} \int_{0}^{+\infty} \frac{3\sin x - \sin 3x}{x^3}dx = \frac{1}{4} \int_{0}^{+\infty} (3\sin x - \sin 3x) d(-\frac{1}{2x^2}) \]

\[= \frac{3}{8} \int_{0}^{+\infty} \frac{\cos x - \cos3x}{x^2} dx = \frac{3}{8} \int_{0}^{+\infty} (\cos x - \cos3x) d(-\frac{1}{x^2}) = \frac{3}{8} \int_{0}^{+\infty} \frac{3 \sin 3x -\sin x}{x} dx \]

迪利克雷积分的结果为:

\[\int_{-\infty}^{+\infty}\frac{\sin x}{x}dx = \pi\]

最终求出\(\displaystyle I = \frac{3}{8}\pi\)

\(\displaystyle I = \int \frac{x^5 - x}{x^8+1} dx\)

\[I = \int \frac{x - \frac{1}{x^3}}{x^4 + \frac{1}{x^4}}dx = \frac{1}{2} \int \frac{1}{x^4 + \frac{1}{x^4}} d(x^2 + \frac{1}{x^2}) \xlongequal{t = x^2 + \frac{1}{x^2}} \frac{1}{2} \int \frac{1}{t^2 - 2}dt = \frac{1}{4\sqrt{2}} \ln | \frac{t - \sqrt{2}}{t + \sqrt{2}} | + C\]

因此\(\displaystyle I = \frac{1}{4\sqrt{2}} \ln |\frac{x^2+\frac{1}{x^2} - \sqrt{2}}{x^2+ \frac{1}{x^2} + \sqrt{2}}| + C = \frac{1}{4\sqrt{2}}\ln |\frac{x^4 - \sqrt{2}x^2 + 1}{x^4 + \sqrt{2}x^2 + 1}| + C\)

\(\displaystyle \int \frac{x + \ln(1 - x)}{x^2} dx\)

\[I = \int \frac{1}{x^2}dx + \int \ln(1-x)d(-\frac{1}{x}) = \int \frac{1}{x} dx -\frac{1}{x}\ln(1-x) + \int\frac{1}{x(x-1)}dx \]

\[= -\frac{1}{x} \ln(1-x) + \int \frac{dx}{x-1} = \frac{x-1}{x}\ln(1-x)dx + C\]

这里特别需要注意求解\(\displaystyle \int \frac{dx}{x-1}\)的结果,事实上结果是\(\displaystyle \ln|x-1| + C\),然而被积函数的定义域为\(x \in (-\infty, 0) \bigcup (0,1)\),因此正确的结果是\(\ln (1-x) + C\)

\(I = \displaystyle \int_{0}^{1}\frac{\ln(1+x)}{1+x^2}dx\)

\[I \xlongequal{x = \tan t} \int_{0}^{\frac{\pi}{4}}\ln(1+\tan t)dt = \int_{0}^{\frac{\pi}{4}}\ln[1+\tan(\frac{\pi}{4}-t)]dt \Rightarrow I = \frac{\pi}{8}\ln 2\]

\(I = \displaystyle \int \frac{xe^{\arctan x}}{(1+x)^{\frac{3}{2}}}dx\)

\[I = \int \frac{x}{\sqrt{1+x^2}}d e^{\arctan x}=\frac{xe^{\arctan x}}{\sqrt{1+x^2}}-\int \frac{e^{\arctan x}}{(1+x^2)^{\frac{3}{2}}}dx = \frac{(x-1)e^{\arctan x}}{(1+x^2)^{\frac{3}{2}}}-I\]

最终得到\(\displaystyle I = \frac{(x-1)e^{\arctan x}}{2(1+x^2)^{\frac{3}{2}}} + C\)

\(I = \displaystyle \int e^{-\frac{x}{2}}\frac{\cos x - \sin x}{\sqrt{\sin x}}dx\)

\[I = 2\int e^{-\frac{x}{2}}d\sqrt{\sin x} - \int e^{-\frac{x}{2}}\sqrt{\sin x}dx = 2e^{-\frac{x}{2}}\sqrt{\sin x} + C\]

\(I = \displaystyle \int \frac{e^{-\sin x}\sin 2x}{\sin^4(\frac{\pi}{4}-\frac{x}{2})} dx\)

\[I = 8\int \frac{e^{-\sin x}\sin x \cos x}{(1-\sin x)^2}dx \xlongequal{t = \sin x}8\int \frac{te^{-t}}{(1-t)^2}dt\]

\[\int \frac{te^{-t}}{(1-t)^2}dt = \int te^{-t}d\frac{1}{1-t} = \frac{te^{-t}}{1-t}-\int e^{-t}dt = \frac{e^{-t}}{1-t} +C_1\]

所以\(I = \displaystyle \frac{8e^{-\sin x}}{1-\sin x} +C\)。这道题目的启发是,分部积分法凑微分时不一定都优先凑指数函数,指数函数与分式相乘时可以考虑对分式凑微分

\(\displaystyle I = \int\frac{(1-2x)e^{\arctan x}}{(1+x^2)}dx\)

\[I = \int\frac{1}{1+x^2}de^{\arctan x} - \int \frac{2xe^{\arctan x}}{(1+x^2)^2}dx = \frac{e^{\arctan x}}{1+x^2} + C\]

\(I=\displaystyle\int_{0}^{+\infty}xe^{-3x}\cos 2xdx\)

考虑一般形式的积分\(J = \displaystyle\int_{0}^{+\infty}x^ne^{-ax}\cos kxdx(a>0,n\in \text{N})\),注意到可以利用 Laplace 变换的微分性质求解:

\[F(s)=\mathcal{L}[x^n\cos kx] = (-1)^n\frac{\partial^n}{\partial s^n}\frac{s}{s^2+k^2}\]

代入题设条件得到\(I=\displaystyle \frac{5}{169}\)。事实上,有如下等式:

\[\frac{\partial^n}{\partial s^n}\frac{s}{s^2+k^2} = \frac{(-1)^nn!}{(s^2+k^2)^{n+1}}\text{Re}[(s+ik)^{n+1}]\] 利用这个等式可以将求导运算转化为求展开系数的问题,计算量有所减少,证明如下: - \(n=0\)时,原命题显然成立; - 假设\(n=l \geq 0\)时,原命题依然成立,那么\(n = l+1\)时有:

\[\frac{\partial^n}{\partial s^n}\frac{s}{s^2+k^2} = (-1)^ll! \cdot \text{Re}[\frac{\partial}{\partial s}\frac{(s+ik)^{l+1}}{(s^2+k^2)^{l+1}}]= \frac{(-1)^{l+1}(l+1)!}{(s^2+k^2)^{(l+1)+1}}\text{Re}[(s+ik)^{(l+1)+1}]\] 证毕,最终得到:

\[F(s)=\mathcal{L}[x^n\cos kx] = (-1)^n\frac{\partial^n}{\partial s^n}\frac{s}{s^2+k^2}=\frac{n!}{(s^2+k^2)^{n+1}}\text{Re}[(s+ik)^{n+1}]\]

\(\displaystyle I = \int_{0}^{+\infty}\frac{x^2}{x^4+x^2+1}dx\)

留数法

求方程\(z^4+z^2+1=0\),得到如下\(4\)个根:

\[z_1=e^{\frac{\pi}{3}i},z_2=e^{\frac{2\pi}{3}i},z_3=e^{\frac{4\pi}{3}i},z_4=e^{\frac{5\pi}{3}i}\] 其中实部为正的有\(z_1\)\(z_2\),因此:

\[I = 2\pi i \sum_{k=1}^{2}\text{Res}[\frac{z^2}{z^4+z^2+1},z_k]\] 下面分别求两个留数:

\[\text{Res}[\frac{z^2}{z^4+z^2+1},z_1]=\frac{z^2_1}{(z_1-z_2)(z_1-z_3)(z_1-z_4)} = \frac{\sqrt{3}i}{6}e^{\frac{4\pi i}{3}}\]

\[\text{Res}[\frac{z^2}{z^4+z^2+1},z_1]=\frac{z^2_2}{(z_2-z_1)(z_2-z_3)(z_2-z_4)} = \frac{\sqrt{3}i}{6}e^{\frac{2\pi i}{3}}\] 因此\(I = \displaystyle\frac{\pi}{2\sqrt{3}}\)。事实上,求留数的分母时,利用好复数的几何性质可以极大地简化运算,注意利用好\(e^{i(\theta + \pi)}=-e^{i\theta}\)这个性质。

倒代换

不难发现如下等式:

\[\int_{0}^{+\infty}\frac{x^2}{x^4+x^2+1}dx \xlongequal{x = \frac{1}{t}}\int_{0}^{+\infty}\frac{1}{t^4+t^2+1}dt=\int_{0}^{+\infty}\frac{1}{x^4+x^2+1}dx\]

那么:

\[I = \frac{1}{2} \int_{0}^{+\infty}\frac{d(x-\frac{1}{x})}{(x-\frac{1}{x})^2+3}=\frac{1}{2\sqrt{3}}[\arctan(\frac{x-\frac{1}{x}}{\sqrt{3}})]_{0}^{+\infty}=\frac{\pi}{2\sqrt{3}}\]

Wallis 公式推导

公式1

Wallis 公式1为:

\[I_n =\int_{0}^{\frac{\pi}{2}}\sin^n xdx = \int_{0}^{\frac{\pi}{2}}\cos^n xdx=\left\{\begin{matrix} \displaystyle\frac{(n-1)!!}{n!!}\cdot\frac{\pi}{2}, n \text{为偶数} \\ \displaystyle\frac{(n-1)!!}{n!!}, \text{else} \end{matrix}\right.(n\in\text{N})\] 不难得到\(I(0)=\displaystyle\frac{\pi}{2}\)\(I(1) = 1\),利用分部积分法有:

\[I_n = -\int_{0}^{\frac{\pi}{2}}\sin^{n-1}xd\cos x=(n-1)\int_{0}^{\frac{\pi}{2}}\cos^2 x \sin^{n-2}xdx\Rightarrow I_n=\frac{n-1}{n}I_{n-2}\] 利用递推式和初始条件最终得到 Wallis 公式,利用区间再现可以得到:

\[\int_0^\frac{\pi}{2}\sin^nxdx=\int_0^\frac{\pi}{2}\cos^nxdx\]

公式2

Wallis 公式2为:

\[I_{n,m} =\int_{0}^{\frac{\pi}{2}}\sin^m x\cos^nxdx =\left\{\begin{matrix} \displaystyle\frac{(m-1)!!(n-1)!!}{(m+n)!!}\cdot\frac{\pi}{2}, m\text{和}n \text{都为偶数} \\ \displaystyle\frac{(m-1)!!(n-1)!!}{(m+n)!!}, \text{else} \end{matrix}\right.(m,n\in\text{N})\] 不难得到\(\displaystyle I_{0,0} = \frac{\pi}{2},I_{0,1}=I_{1,0} =I_{1,1} = 1\),下面推导积分的递推条件:

\[I_{n,m} = \frac{1}{m+1}\int_{0}^{\frac{\pi}{2}}\cos^{n-1}xd \sin^{m+1} x= \frac{n-1}{m+1}\int_{0}^{\frac{\pi}{2}}\sin^{m+2}x\cos^{n-2}xdx\]

\[=\frac{n-1}{m+1}\int_{0}^{\frac{\pi}{2}}\sin^{m}x\cos^{n-2}(1-\cos^2 x)xdx\Rightarrow I_{m,n}=\frac{n-1}{m+n}I_{m,n-2}\] 类似地有\(\displaystyle I_{m,n}=\frac{m-1}{m+n}I_{m-2,n}\),利用递推式和初始条件可以得到原结论。注意\(0!!=(-1)!!=1\)

\(I=\displaystyle\int_{-\infty}^{+\infty}e^{x-\sinh^2 x}dx\)

首先证明,\(a>0,b>0\)时,\(\displaystyle\int_{0}^{+\infty}f[(ax-\frac{b}{x})^2]dx=\frac{1}{a}\int_{0}^{+\infty}f(x^2)dx\)

\[\int_{0}^{+\infty}f[(ax-\frac{b}{x})^2]dx\xlongequal{u=\frac{b}{ax}}\int_{0}^{+\infty}\frac{b}{au^2}f[(au-\frac{b}{u})^2]du\]

\[\Rightarrow \int_{0}^{+\infty}f[(ax-\frac{b}{x})^2]dx = \frac{1}{2a}\int_{0}^{+\infty}f[(ax-\frac{b}{x})^2]d(ax-\frac{b}{x})=\frac{1}{a}\int_{0}^{+\infty}f(x^2)dx\] 注意换元时的积分限。利用这个结论可计算原积分:

\[I=\int_{-\infty}^{+\infty}e^{-(\frac{e^x}{2}-\frac{e^{-x}}{2})^2}de^x\xlongequal{t=e^x}\int_{0}^{+\infty}e^{-(\frac{t}{2}-\frac{1}{2t})^2}dt=2\int_{0}^{+\infty}e^{-t^2}dt=\sqrt{\pi}\]

\(\displaystyle \int\frac{dx}{1+x^n}\),其中\(n=3,4\)

\(I=\displaystyle \int\frac{dx}{1+x^3}\)

\(\displaystyle \frac{1}{1+x^3}=\frac{A}{x+1}+\frac{Bx+C}{x^2-x+1}\),利用留数法的思想,可以得到如下方程:

\[\frac{1}{x^2-x+1}=A+\frac{(Bx+C)(x+1)}{x^2-x+1}\Rightarrow A = \frac{1}{3}\] 再令\(x= 1\)\(x=0\),得到如下两个方程:

\[A+C=1\]

\[\frac{A}{2}+B+C=\frac{1}{2}\] 解得:

\[\left\{\begin{matrix} \displaystyle A = \frac{1}{3}\\ \displaystyle B=-\frac{1}{3}\\ \displaystyle C=\frac{2}{3} \end{matrix}\right.\] 那么\(I=\displaystyle \frac{1}{3}\ln\frac{|x+1|}{\sqrt{x^2-x+1}}+\frac{1}{\sqrt{3}}\arctan(\frac{2x-1}{\sqrt{3}})+C\)

\(I=\displaystyle \int\frac{dx}{1+x^4}\)

\(x^4+1=(x^2+1)^2-2x^2=(x^2-\sqrt{2}x+1)(x^2+\sqrt{2}x+1)\),有:

\[\frac{1}{1+x^4}=\frac{Ax+B}{x^2-\sqrt{2}x+1}+\frac{Cx+D}{x^2+\sqrt{2}x+1}\] 考虑到\(x^4+1=0\)没有实根,并且复根形式上较为复杂,直接赋值求系数:

\[\left\{\begin{matrix} \displaystyle \sqrt{2}A+B+\frac{\sqrt{2}}{5}C+\frac{1}{5}D=\frac{1}{5}(\text{令}x=\sqrt2) \\ \displaystyle -\frac{\sqrt 2}{5}A+\frac{1}{5}B-\sqrt{2}C+D =\frac{1}{5} (\text{令}x=-\sqrt2) \\ \displaystyle B+D=1 (\text{令}x=0) \\ \displaystyle \sqrt{2}A+ 2B+\frac{\sqrt{2}}{5}C+ \frac{2}{5}D =\frac{4}{5}(\text{令}x=\frac{\sqrt{2}}{2}) \end{matrix}\right.\] 解得: \[\left\{\begin{matrix} \displaystyle A = -\frac{\sqrt{2}}{4} \\ \displaystyle B = \frac{1}{2} \\ \displaystyle C = \frac{\sqrt{2}}{4}\\ \displaystyle D = \frac{1}{2} \end{matrix}\right.\] 计算过程略,求得\(\displaystyle I = \frac{\sqrt{2}}{8}\ln\frac{x^2+\sqrt{2}x+1}{x^2-\sqrt{2}x+1}+\frac{\sqrt{2}}{4}\arctan(\sqrt{2}x-1)+\frac{\sqrt{2}}{4}\arctan(\sqrt{2}x+1)+C\)

评论